Discontinuity of velocity in a problem on mass flow and momentum

In summary: You'll notice that's the same equation as before, for variable mass systems. It includes the ##\vec{v}_{rel}\frac{dm}{dt}## term that you would not get if you erroneously applied NII to the variable mass...
  • #36
etotheipi said:

They claim

When F is zero, in particular, Equation (2) implies that the particle will remain at rest in a system where it is originally at rest, but it will be accelerated by the "force" -v dm/dt in a system where the particle moves with the velocity v!

Let's check that:

Equation (2) results in the acceleration

##\dot v = \frac{{F - \dot m \cdot v}}{m}##

With Galiean transformation we have

##v' = v - u##
##\dot v' = \dot v##
##m' = m##
##\dot m' = \dot m##

and due to the principle of relativity also

##F' = \dot m' \cdot v' + m' \cdot \dot v' = F - \dot m \cdot u##

That means for the acceleration resulting from (2)

##\dot v' = \frac{{F' - \dot m' \cdot v'}}{{m'}} = \dot v##

Everything remains consistent (obviously). The resulting acceleration is frame-independent as expected. If the particle is at rest in one frame then there is no other frame where it is accelerated. Thus, the claim above appears to be wrong. That makes the paper pointless.

etotheipi said:
As for the mine-cart, there is no force acting on the leaving sand.

If you split mine-cart and sand into an open system containing the mine-cart and the sand that did not yet leave the cart and another open systems containing the sand that already leaved the cart than the momentums of both systems are changing (if the cart has non-zero velocity). With F=dp/dt this means that there are forces acting on both systems.

etotheipi said:
And this is a frame independent observation.

Acceleration, mass and mass transfer are frame-independent. Velocity, momentum and F=dp/dt are not.
 
  • Skeptical
  • Sad
Likes etotheipi and Motore
Physics news on Phys.org
  • #37
etotheipi said:
Now, if you were to apply your equation, ##\vec{F} = \vec{0} = \dot{m}\vec{v} + m\dot{\vec{v}}##, to the mine-cart

F=dp/dt is frame-dependent! If it is zero in the rest-frame of the cart than it is not zero in a frame of reference where the cart is moving with v>0. You again made the mistake to use F=dp/dt for an open system together with an assumption that holds for closed systems only.
 
  • Skeptical
Likes davenn
  • #38
To be honest, I really have a hard time trying to interpret what you are saying.

DrStupid said:
If you split mine-cart and sand into an open system containing the mine-cart and the sand that did not yet leave the cart and another open systems containing the sand that already leaved the cart than the momentums of both systems are changing (if the cart has non-zero velocity). With F=dp/dt this means that there are forces acting on both systems.

DrStupid said:
F=dp/dt is frame-dependent! If it is zero in the rest-frame of the cart than it is not zero in a frame of reference where the cart is moving with v>0.

This is completely confused. If the external force on the 'mine-cart + me holding the ball bearings' system in my example in #35 is non-zero, then what is the physical origin of this supposedly present force?
 
  • #39
vanhees71 said:
If the external force on the 'mine-cart + me holding the ball bearings' system in my example in #35 is non-zero, then what is the physical origin of this supposedly present force?

It comes from the ball bearings that leaved the system. It is exactly the same situation as with the sand: You have an open system containing the cart, you and some mass (e.g. sand, ball bearings or whatever) and you have another open system containing the exhausted mass that already dropped out of the cart. There is a mass transfer ##\dot m## from one system to the other. If the systems are moving with ##v## than there is also a momentum transfer ##\dot m \cdot v## because the exchanged mass carries its momentum from one system to the other. According to F=dp/dt the momentum transfer corresponds to a force between the two systems. The cart system exerts the force ##\dot m \cdot v## on the exhaust system and the exhaust system exterts the corresponding counter force ##-\dot m \cdot v## on the cart system.

Maybe you don't understand it because you expect something difficult. But it is in fact very trivial.
 
  • Skeptical
Likes davenn, weirdoguy and etotheipi
  • #40
DrStupid said:
According to F=dp/dt the momentum transfer corresponds to a force between the two systems.

No, that is wrong. The correct answer is that 'there is no such force'. You can't just magic up a force from somewhere, real forces arise due to physical interactions and there is no such physical interaction that you can identify in my example.

You are sitting in the mine-cart, and you drop the ball bearings from rest through the hole in the floor at a rate of one ball per second. You impart no horizontal impulse to the ball bearings, and they impart no horizontal impulse to you.

The point is that you can obtain a change in momentum without a force, due to mass leaving the system. That was the whole point of my example, to explain that naively applying ##\vec{F} = \frac{d\vec{p}}{dt}## to a variable mass system (like the 'mine-cart + me with ball bearings' system) gives contradictory results.

You have a seriously flawed interpretation of the equation ##\vec{F} = \dot{\vec{p}}##, and your vague and deflective responses only go to show that you cannot justify your position.

Please, please, consult any fluid dynamics textbook that deals with control volumes, or any introductory mechanics textbook that deals with variable mass systems. It will become clear to you that the only fully consistent picture is that ##\vec{F} = \dot{\vec{p}}## can only be applied to systems of constant mass.
 
  • Like
Likes nasu, davenn and weirdoguy
  • #41
etotheipi said:
real forces arise due to physical interactions and there is no such physical interaction that you can identify in my example.

Are you really telling me that mass transfer is no physical interaction? That is ridiculous.

etotheipi said:
You impart no horizontal impulse to the ball bearings, and they impart no horizontal impulse to you.

I didn't claim that there is a force between me and the ball bearings. In #39 I defined the systems that I am talking about. Can I at least expect you to read my postings before you reply?

etotheipi said:
The point is that you can obtain a change in momentum without a force, due to mass leaving the system.

Not with F=dp/dt. Just for the case that you don't understand it: It means that the force F is equal to the change of momentum p with time t. With this relationship there is no change of momentum if there is no force. If you come to another conclusion than you did something wrong.
 
  • Skeptical
Likes etotheipi
  • #42
Thread is closed for Moderation...
 
  • Like
Likes vanhees71
  • #43
PeterDonis said:
how is force defined when the system is not of constant mass?

To wrap up the discussion in this thread, I'm going to answer this question and then give some brief follow-up. The total force on the spacecraft in the scenario given in the OP of this thread is

$$
\vec{F} = \vec{F}_\text{external} + \left( \vec{u} - \vec{v} \right) \frac{dm}{dt} = m \frac{d \vec{v}}{dt} = m \vec{a}
$$

The total force here is frame invariant and has the obvious physical interpretation of the force exerted by the dust on the spacecraft , plus the "magical" external force exerted to oppose the dust force and keep the spacecraft moving uniformly (i.e., so that ##\vec{a} = 0##).

This also shows that Newton's Second Law, in the form ##\vec{F} = m \vec{a}##, with the total force ##\vec{F}## properly defined, works just fine for systems with time-varying mass.

What about the equation ##\vec{F} = d \vec{p} / dt##? We can obtain that from the above equation by rearranging it as follows:

$$
\vec{F}_\text{external} + \vec{u} \frac{dm}{dt} = m \frac{d \vec{v}}{dt} + \vec{v} \frac{dm}{dt} = \frac{d \vec{p}}{dt}
$$

The LHS is the sum of a frame invariant term and a frame-dependent term, so the LHS as a whole is frame-dependent; thus, the RHS must also be frame-dependent. So this equation, unlike the equation above, does not express a relationship between frame-invariant quantities.

Furthermore, the "force" appearing on the LHS of this equation is not the total force, since the term ##\vec{v} \, d \vec{m} / dt##, which is part of the total force, at least if "total force" has the obvious physical meaning described above, is included in the RHS, not the LHS. Defining the LHS of this equation as "force" is giving a different, frame-dependent meaning to the term "force".

It is true that, mathematically, both equations are equivalent in terms of what they will say about actual experimental predictions, and what one chooses to mean by the term "force" is a question of choice of words, not physics. The first equation seems to me to be a clearer representation of the actual physics, since it relates invariant quantities. But if someone wants to use the second equation, as long as they are able to get the right answers with it, it's up to them.
 
  • Like
Likes nasu, vanhees71, DrStupid and 2 others
  • #44
And with that, this thread will remain closed.
 
  • Like
Likes vanhees71, etotheipi and berkeman

Similar threads

Replies
17
Views
943
  • Mechanics
Replies
30
Views
809
Replies
35
Views
3K
Replies
3
Views
799
  • Advanced Physics Homework Help
Replies
6
Views
304
Replies
3
Views
792
Replies
5
Views
1K
Replies
0
Views
1K
  • Introductory Physics Homework Help
Replies
12
Views
201
Back
Top